A series whose convergence is equivalent to the Riemann hypothesis












3












$begingroup$


It was claimed here that the convergence of the series$$sum_{n=2}^infty frac{Lambda(n)-1}{n^{1/2}log^3 n}tag1$$(where $Lambda$ is the Von Mangoldt function) is equivalent to the Riemann hypothesis. Is this true? That post provided a link to the Wikipedia article about the Von Mangoldt function, which does not mention this. Also, this page about the Von Mangoldt function in the context of the Riemann hypothesis makes no mention to that.



If it is true that the convergence of the series $(1)$ is equivalent to the Riemann hypothesis, then I would like to have a reference for that.










share|cite|improve this question









$endgroup$

















    3












    $begingroup$


    It was claimed here that the convergence of the series$$sum_{n=2}^infty frac{Lambda(n)-1}{n^{1/2}log^3 n}tag1$$(where $Lambda$ is the Von Mangoldt function) is equivalent to the Riemann hypothesis. Is this true? That post provided a link to the Wikipedia article about the Von Mangoldt function, which does not mention this. Also, this page about the Von Mangoldt function in the context of the Riemann hypothesis makes no mention to that.



    If it is true that the convergence of the series $(1)$ is equivalent to the Riemann hypothesis, then I would like to have a reference for that.










    share|cite|improve this question









    $endgroup$















      3












      3








      3


      4



      $begingroup$


      It was claimed here that the convergence of the series$$sum_{n=2}^infty frac{Lambda(n)-1}{n^{1/2}log^3 n}tag1$$(where $Lambda$ is the Von Mangoldt function) is equivalent to the Riemann hypothesis. Is this true? That post provided a link to the Wikipedia article about the Von Mangoldt function, which does not mention this. Also, this page about the Von Mangoldt function in the context of the Riemann hypothesis makes no mention to that.



      If it is true that the convergence of the series $(1)$ is equivalent to the Riemann hypothesis, then I would like to have a reference for that.










      share|cite|improve this question









      $endgroup$




      It was claimed here that the convergence of the series$$sum_{n=2}^infty frac{Lambda(n)-1}{n^{1/2}log^3 n}tag1$$(where $Lambda$ is the Von Mangoldt function) is equivalent to the Riemann hypothesis. Is this true? That post provided a link to the Wikipedia article about the Von Mangoldt function, which does not mention this. Also, this page about the Von Mangoldt function in the context of the Riemann hypothesis makes no mention to that.



      If it is true that the convergence of the series $(1)$ is equivalent to the Riemann hypothesis, then I would like to have a reference for that.







      sequences-and-series reference-request riemann-hypothesis






      share|cite|improve this question













      share|cite|improve this question











      share|cite|improve this question




      share|cite|improve this question










      asked Jan 21 at 14:13









      José Carlos SantosJosé Carlos Santos

      165k22132235




      165k22132235






















          2 Answers
          2






          active

          oldest

          votes


















          3












          $begingroup$

          Claymath official description of the Riemann hypothesis claims the RH is true iff $pi(x) = Li(x)+O(x^{1/2}log x)$ so $psi(x) = x+O(x^{1/2}log^2 x)$ and I have been quite sloppy in that it implies the RH is true iff $$sum_{n=2}^infty frac{Lambda(n)-1}{n^{1/2}log^{color{red}{3+epsilon}} n} < inftytag{1}$$



          as with partial summation $$sum_{n le x} (Lambda(n)-1) frac{1}{n^{1/2}log^{a} n}=frac{psi(x)-x}{x^{1/2}log^a x}+sum_{n le x-1} (psi(n)-n)O(frac{1}{n^{3/2}log^a n})\ = O(log^{2-a}(x))+sum_{n le x} O(frac{1}{nlog^{a-2} n})$$





          The point is to show an effective explicit formula (p.28) $$psi(x) =sum_{n le x} Lambda(n)= x - sum_{|Im(rho)| le T} frac{x^{rho}}{rho}+O(frac{xlog^2 x}{T})=x - sum_{kle K} 2Re(frac{x^{rho_k}}{rho_k})+O(frac{xlog^2 x}{K/log K})$$
          where $K = N(T)$ and the density of zeros gives $K sim C T log T,T sim c K/log K$,$Im(rho_k) sim c k/log k$.



          In this form, under the RH, with $K = x^{1/2}$ it yields
          $$psi(x) =x +O(x^{1/2}log^{2+delta})$$



          Plotting those things indicates the series may converge very slowly with $epsilon = 0$ and it is quite certain (under the RH) it converges with $epsilon = 2$.






          share|cite|improve this answer











          $endgroup$









          • 1




            $begingroup$
            Then I suggest that you delete this answer of yours, since it is not correct.
            $endgroup$
            – José Carlos Santos
            Jan 21 at 18:07



















          1












          $begingroup$

          A good place to start is by reading Terry Tao's post "The Riemann hypothesis in various settings"






          share|cite|improve this answer









          $endgroup$













          • $begingroup$
            so (if we can't find a cancellation in the phases of the explicit formula) we really need $log^{3+epsilon} n$ instead of $log^3 n$
            $endgroup$
            – reuns
            Jan 21 at 18:25











          Your Answer





          StackExchange.ifUsing("editor", function () {
          return StackExchange.using("mathjaxEditing", function () {
          StackExchange.MarkdownEditor.creationCallbacks.add(function (editor, postfix) {
          StackExchange.mathjaxEditing.prepareWmdForMathJax(editor, postfix, [["$", "$"], ["\\(","\\)"]]);
          });
          });
          }, "mathjax-editing");

          StackExchange.ready(function() {
          var channelOptions = {
          tags: "".split(" "),
          id: "69"
          };
          initTagRenderer("".split(" "), "".split(" "), channelOptions);

          StackExchange.using("externalEditor", function() {
          // Have to fire editor after snippets, if snippets enabled
          if (StackExchange.settings.snippets.snippetsEnabled) {
          StackExchange.using("snippets", function() {
          createEditor();
          });
          }
          else {
          createEditor();
          }
          });

          function createEditor() {
          StackExchange.prepareEditor({
          heartbeatType: 'answer',
          autoActivateHeartbeat: false,
          convertImagesToLinks: true,
          noModals: true,
          showLowRepImageUploadWarning: true,
          reputationToPostImages: 10,
          bindNavPrevention: true,
          postfix: "",
          imageUploader: {
          brandingHtml: "Powered by u003ca class="icon-imgur-white" href="https://imgur.com/"u003eu003c/au003e",
          contentPolicyHtml: "User contributions licensed under u003ca href="https://creativecommons.org/licenses/by-sa/3.0/"u003ecc by-sa 3.0 with attribution requiredu003c/au003e u003ca href="https://stackoverflow.com/legal/content-policy"u003e(content policy)u003c/au003e",
          allowUrls: true
          },
          noCode: true, onDemand: true,
          discardSelector: ".discard-answer"
          ,immediatelyShowMarkdownHelp:true
          });


          }
          });














          draft saved

          draft discarded


















          StackExchange.ready(
          function () {
          StackExchange.openid.initPostLogin('.new-post-login', 'https%3a%2f%2fmath.stackexchange.com%2fquestions%2f3081918%2fa-series-whose-convergence-is-equivalent-to-the-riemann-hypothesis%23new-answer', 'question_page');
          }
          );

          Post as a guest















          Required, but never shown

























          2 Answers
          2






          active

          oldest

          votes








          2 Answers
          2






          active

          oldest

          votes









          active

          oldest

          votes






          active

          oldest

          votes









          3












          $begingroup$

          Claymath official description of the Riemann hypothesis claims the RH is true iff $pi(x) = Li(x)+O(x^{1/2}log x)$ so $psi(x) = x+O(x^{1/2}log^2 x)$ and I have been quite sloppy in that it implies the RH is true iff $$sum_{n=2}^infty frac{Lambda(n)-1}{n^{1/2}log^{color{red}{3+epsilon}} n} < inftytag{1}$$



          as with partial summation $$sum_{n le x} (Lambda(n)-1) frac{1}{n^{1/2}log^{a} n}=frac{psi(x)-x}{x^{1/2}log^a x}+sum_{n le x-1} (psi(n)-n)O(frac{1}{n^{3/2}log^a n})\ = O(log^{2-a}(x))+sum_{n le x} O(frac{1}{nlog^{a-2} n})$$





          The point is to show an effective explicit formula (p.28) $$psi(x) =sum_{n le x} Lambda(n)= x - sum_{|Im(rho)| le T} frac{x^{rho}}{rho}+O(frac{xlog^2 x}{T})=x - sum_{kle K} 2Re(frac{x^{rho_k}}{rho_k})+O(frac{xlog^2 x}{K/log K})$$
          where $K = N(T)$ and the density of zeros gives $K sim C T log T,T sim c K/log K$,$Im(rho_k) sim c k/log k$.



          In this form, under the RH, with $K = x^{1/2}$ it yields
          $$psi(x) =x +O(x^{1/2}log^{2+delta})$$



          Plotting those things indicates the series may converge very slowly with $epsilon = 0$ and it is quite certain (under the RH) it converges with $epsilon = 2$.






          share|cite|improve this answer











          $endgroup$









          • 1




            $begingroup$
            Then I suggest that you delete this answer of yours, since it is not correct.
            $endgroup$
            – José Carlos Santos
            Jan 21 at 18:07
















          3












          $begingroup$

          Claymath official description of the Riemann hypothesis claims the RH is true iff $pi(x) = Li(x)+O(x^{1/2}log x)$ so $psi(x) = x+O(x^{1/2}log^2 x)$ and I have been quite sloppy in that it implies the RH is true iff $$sum_{n=2}^infty frac{Lambda(n)-1}{n^{1/2}log^{color{red}{3+epsilon}} n} < inftytag{1}$$



          as with partial summation $$sum_{n le x} (Lambda(n)-1) frac{1}{n^{1/2}log^{a} n}=frac{psi(x)-x}{x^{1/2}log^a x}+sum_{n le x-1} (psi(n)-n)O(frac{1}{n^{3/2}log^a n})\ = O(log^{2-a}(x))+sum_{n le x} O(frac{1}{nlog^{a-2} n})$$





          The point is to show an effective explicit formula (p.28) $$psi(x) =sum_{n le x} Lambda(n)= x - sum_{|Im(rho)| le T} frac{x^{rho}}{rho}+O(frac{xlog^2 x}{T})=x - sum_{kle K} 2Re(frac{x^{rho_k}}{rho_k})+O(frac{xlog^2 x}{K/log K})$$
          where $K = N(T)$ and the density of zeros gives $K sim C T log T,T sim c K/log K$,$Im(rho_k) sim c k/log k$.



          In this form, under the RH, with $K = x^{1/2}$ it yields
          $$psi(x) =x +O(x^{1/2}log^{2+delta})$$



          Plotting those things indicates the series may converge very slowly with $epsilon = 0$ and it is quite certain (under the RH) it converges with $epsilon = 2$.






          share|cite|improve this answer











          $endgroup$









          • 1




            $begingroup$
            Then I suggest that you delete this answer of yours, since it is not correct.
            $endgroup$
            – José Carlos Santos
            Jan 21 at 18:07














          3












          3








          3





          $begingroup$

          Claymath official description of the Riemann hypothesis claims the RH is true iff $pi(x) = Li(x)+O(x^{1/2}log x)$ so $psi(x) = x+O(x^{1/2}log^2 x)$ and I have been quite sloppy in that it implies the RH is true iff $$sum_{n=2}^infty frac{Lambda(n)-1}{n^{1/2}log^{color{red}{3+epsilon}} n} < inftytag{1}$$



          as with partial summation $$sum_{n le x} (Lambda(n)-1) frac{1}{n^{1/2}log^{a} n}=frac{psi(x)-x}{x^{1/2}log^a x}+sum_{n le x-1} (psi(n)-n)O(frac{1}{n^{3/2}log^a n})\ = O(log^{2-a}(x))+sum_{n le x} O(frac{1}{nlog^{a-2} n})$$





          The point is to show an effective explicit formula (p.28) $$psi(x) =sum_{n le x} Lambda(n)= x - sum_{|Im(rho)| le T} frac{x^{rho}}{rho}+O(frac{xlog^2 x}{T})=x - sum_{kle K} 2Re(frac{x^{rho_k}}{rho_k})+O(frac{xlog^2 x}{K/log K})$$
          where $K = N(T)$ and the density of zeros gives $K sim C T log T,T sim c K/log K$,$Im(rho_k) sim c k/log k$.



          In this form, under the RH, with $K = x^{1/2}$ it yields
          $$psi(x) =x +O(x^{1/2}log^{2+delta})$$



          Plotting those things indicates the series may converge very slowly with $epsilon = 0$ and it is quite certain (under the RH) it converges with $epsilon = 2$.






          share|cite|improve this answer











          $endgroup$



          Claymath official description of the Riemann hypothesis claims the RH is true iff $pi(x) = Li(x)+O(x^{1/2}log x)$ so $psi(x) = x+O(x^{1/2}log^2 x)$ and I have been quite sloppy in that it implies the RH is true iff $$sum_{n=2}^infty frac{Lambda(n)-1}{n^{1/2}log^{color{red}{3+epsilon}} n} < inftytag{1}$$



          as with partial summation $$sum_{n le x} (Lambda(n)-1) frac{1}{n^{1/2}log^{a} n}=frac{psi(x)-x}{x^{1/2}log^a x}+sum_{n le x-1} (psi(n)-n)O(frac{1}{n^{3/2}log^a n})\ = O(log^{2-a}(x))+sum_{n le x} O(frac{1}{nlog^{a-2} n})$$





          The point is to show an effective explicit formula (p.28) $$psi(x) =sum_{n le x} Lambda(n)= x - sum_{|Im(rho)| le T} frac{x^{rho}}{rho}+O(frac{xlog^2 x}{T})=x - sum_{kle K} 2Re(frac{x^{rho_k}}{rho_k})+O(frac{xlog^2 x}{K/log K})$$
          where $K = N(T)$ and the density of zeros gives $K sim C T log T,T sim c K/log K$,$Im(rho_k) sim c k/log k$.



          In this form, under the RH, with $K = x^{1/2}$ it yields
          $$psi(x) =x +O(x^{1/2}log^{2+delta})$$



          Plotting those things indicates the series may converge very slowly with $epsilon = 0$ and it is quite certain (under the RH) it converges with $epsilon = 2$.







          share|cite|improve this answer














          share|cite|improve this answer



          share|cite|improve this answer








          edited Jan 22 at 12:58

























          answered Jan 21 at 18:04









          reunsreuns

          21.1k21250




          21.1k21250








          • 1




            $begingroup$
            Then I suggest that you delete this answer of yours, since it is not correct.
            $endgroup$
            – José Carlos Santos
            Jan 21 at 18:07














          • 1




            $begingroup$
            Then I suggest that you delete this answer of yours, since it is not correct.
            $endgroup$
            – José Carlos Santos
            Jan 21 at 18:07








          1




          1




          $begingroup$
          Then I suggest that you delete this answer of yours, since it is not correct.
          $endgroup$
          – José Carlos Santos
          Jan 21 at 18:07




          $begingroup$
          Then I suggest that you delete this answer of yours, since it is not correct.
          $endgroup$
          – José Carlos Santos
          Jan 21 at 18:07











          1












          $begingroup$

          A good place to start is by reading Terry Tao's post "The Riemann hypothesis in various settings"






          share|cite|improve this answer









          $endgroup$













          • $begingroup$
            so (if we can't find a cancellation in the phases of the explicit formula) we really need $log^{3+epsilon} n$ instead of $log^3 n$
            $endgroup$
            – reuns
            Jan 21 at 18:25
















          1












          $begingroup$

          A good place to start is by reading Terry Tao's post "The Riemann hypothesis in various settings"






          share|cite|improve this answer









          $endgroup$













          • $begingroup$
            so (if we can't find a cancellation in the phases of the explicit formula) we really need $log^{3+epsilon} n$ instead of $log^3 n$
            $endgroup$
            – reuns
            Jan 21 at 18:25














          1












          1








          1





          $begingroup$

          A good place to start is by reading Terry Tao's post "The Riemann hypothesis in various settings"






          share|cite|improve this answer









          $endgroup$



          A good place to start is by reading Terry Tao's post "The Riemann hypothesis in various settings"







          share|cite|improve this answer












          share|cite|improve this answer



          share|cite|improve this answer










          answered Jan 21 at 15:47









          ConradConrad

          67335




          67335












          • $begingroup$
            so (if we can't find a cancellation in the phases of the explicit formula) we really need $log^{3+epsilon} n$ instead of $log^3 n$
            $endgroup$
            – reuns
            Jan 21 at 18:25


















          • $begingroup$
            so (if we can't find a cancellation in the phases of the explicit formula) we really need $log^{3+epsilon} n$ instead of $log^3 n$
            $endgroup$
            – reuns
            Jan 21 at 18:25
















          $begingroup$
          so (if we can't find a cancellation in the phases of the explicit formula) we really need $log^{3+epsilon} n$ instead of $log^3 n$
          $endgroup$
          – reuns
          Jan 21 at 18:25




          $begingroup$
          so (if we can't find a cancellation in the phases of the explicit formula) we really need $log^{3+epsilon} n$ instead of $log^3 n$
          $endgroup$
          – reuns
          Jan 21 at 18:25


















          draft saved

          draft discarded




















































          Thanks for contributing an answer to Mathematics Stack Exchange!


          • Please be sure to answer the question. Provide details and share your research!

          But avoid



          • Asking for help, clarification, or responding to other answers.

          • Making statements based on opinion; back them up with references or personal experience.


          Use MathJax to format equations. MathJax reference.


          To learn more, see our tips on writing great answers.




          draft saved


          draft discarded














          StackExchange.ready(
          function () {
          StackExchange.openid.initPostLogin('.new-post-login', 'https%3a%2f%2fmath.stackexchange.com%2fquestions%2f3081918%2fa-series-whose-convergence-is-equivalent-to-the-riemann-hypothesis%23new-answer', 'question_page');
          }
          );

          Post as a guest















          Required, but never shown





















































          Required, but never shown














          Required, but never shown












          Required, but never shown







          Required, but never shown

































          Required, but never shown














          Required, but never shown












          Required, but never shown







          Required, but never shown







          Popular posts from this blog

          Can a sorcerer learn a 5th-level spell early by creating spell slots using the Font of Magic feature?

          ts Property 'filter' does not exist on type '{}'

          mat-slide-toggle shouldn't change it's state when I click cancel in confirmation window